Buscar

Campo magnético gerado

Calcule o vetor campo magnético devido aos dois fios semi-infinitos da Figura 1 (que conduzem a corrente I) em um ponto arbitrário z > 0 do eixo vertical (perpendicular ao papel) que contém o centro O da espira circular. Sugestão: os dois fios semi-infinitos que conduzem a corrente I são equivalentes a um fio infinito que conduz a corrente I e um fio finito que conduz a corrente –I.

💡 3 Respostas

User badge image

Andre Smaira

Campo magnético gerado

Calcule o vetor campo magnético devido aos dois fios semi-infinitos da Figura 1 (que conduzem a corrente I) em um ponto arbitrário z > 0 do eixo vertical (perpendicular ao papel) que contém o centro O da espira circular. Sugestão: os dois fios semi-infinitos que conduzem a corrente I são equivalentes a um fio infinito que conduz a corrente I e um fio finito que conduz a corrente –I.

#Eletricidade#Eletromagnetismo


Nesse exercício vamos usar a Lei de Biot Savart.


Pela lei citada, temos:

$$\vec B=\int \dfrac{\mu_0I}{4\pi}\dfrac{d\vec l\times \hat r}{r^2}$$

Para o nosso caso a corrente está sempre ao longo do eixo $x$:

$$\vec B=-\hat y\int_{-\infty}^{-R} \dfrac{\mu_0I}{4\pi}\dfrac{dx\sin\theta}{r^2}-\hat y\int_R^\infty \dfrac{\mu_0I}{4\pi}\dfrac{dx\sin\theta}{r^2}$$

Como sugerido no exercício, podemos juntar o cálculo dos dois fios em um e retirar a parte central, que nama mais é que usar a superposição das integrais:

$$\vec B=-\hat y\int_{-\infty}^\infty \dfrac{\mu_0I}{4\pi}\dfrac{dx\sin\theta}{r^2}+\hat y\int_{-R}^R \dfrac{\mu_0I}{4\pi}\dfrac{dx\sin\theta}{r^2}$$

Mas o seno do ângulo entre os vetores é dado por:

$$\sin\theta=\dfrac{z}{\sqrt{x^2+z^2}}$$

Voltando pra integral:

$$\vec B=-\dfrac{\mu_0Iz}{4\pi}\hat y\int_{-\infty}^\infty \dfrac{dx}{(x^2+z^2)^{5/2}}+\dfrac{\mu_0Iz}{4\pi}\hat y\int_{-R}^R \dfrac{dx}{(x^2+z^2)^{5/2}}$$

Mas a integral da esquerda nada mais é que o campo magnético de um fio infinito:

$$\vec B=-\dfrac{\mu_0I}{2\pi z}\hat y+\dfrac{\mu_0Iz}{4\pi}\hat y\int_{-R}^R \dfrac{dx}{(x^2+z^2)^{5/2}}$$

Fazendo $x=z\tan\phi\Rightarrow dx=z\sec^2\phi d\phi$, temos:

$$\vec B=-\dfrac{\mu_0I}{2\pi z}\hat y+\dfrac{\mu_0Iz}{4\pi}\hat y\int_{-\arctan{(R/z)}}^{\arctan{(R/z)}} \dfrac{z\sec^2\phi d\phi}{(z^2\tan^2\phi+z^2)^{5/2}}$$

Do princípio fundamental da trigonometria:

$$\sin^2\phi+\cos^2\phi=1\Rightarrow\tan^2\phi+1=\sec^2\phi$$

Então:

$$\vec B=-\dfrac{\mu_0I}{2\pi z}\hat y+\dfrac{\mu_0Iz^2}{4\pi z^5}\hat y\int_{-\arctan{(R/z)}}^{\arctan{(R/z)}} \dfrac{\sec^2\phi d\phi}{(\sec^2\phi)^{5/2}}=-\dfrac{\mu_0I}{2\pi z}\hat y+\dfrac{\mu_0I}{4\pi z^3}\hat y\int_{-\arctan{(R/z)}}^{\arctan{(R/z)}} \cos^3\phi d\phi$$

Novamente pelo princípio fundamental da trigonometria:

$$\vec B=-\dfrac{\mu_0I}{2\pi z}\hat y+\dfrac{\mu_0I}{4\pi z^3}\hat y\int_{-\arctan{(R/z)}}^{\arctan{(R/z)}} (1-\sin^2\phi)\cos\phi d\phi$$

Fazendo $u=\sin\phi\Rightarrow du=\cos\phi d\phi$:

$$\vec B=-\dfrac{\mu_0I}{2\pi z}\hat y+\dfrac{\mu_0I}{4\pi z^3}\hat y\int_{-\frac{R}{\sqrt{R^2+z^2}}}^{\frac{R}{\sqrt{R^2+z^2}}} (1-u^2)du=-\dfrac{\mu_0I}{2\pi z}\hat y+\dfrac{\mu_0I}{4\pi z^3}\hat y \left[u-\frac{1}{3}u^3\right]_{-\frac{R}{\sqrt{R^2+z^2}}}^{\frac{R}{\sqrt{R^2+z^2}}}$$

Ao substituir os limites, ficamos com:

$$\vec B=-\dfrac{\mu_0I}{2\pi z}\hat y+\dfrac{\mu_0I}{4\pi z^3}\hat y \left[\dfrac{2R}{\sqrt{R^2+z^2}}-\frac{2}{3}\dfrac{R^3}{(R^2+z^2)^{3/2}}\right]$$


Temos, portanto:

$$\boxed{\vec B=\hat y\dfrac{\mu_0I}{2\pi z}\left[\dfrac{1}{3z^2}\dfrac{R}{(R^2+z^2)^{3/2}} \left(2R^2+3z^2\right)-1\right]}$$

0
Dislike0
User badge image

Andre Smaira

Nesse exercício vamos usar a Lei de Biot Savart.


Pela lei citada, temos:

$$\vec B=\int \dfrac{\mu_0I}{4\pi}\dfrac{d\vec l\times \hat r}{r^2}$$

Para o nosso caso a corrente está sempre ao longo do eixo $x$:

$$\vec B=-\hat y\int_{-\infty}^{-R} \dfrac{\mu_0I}{4\pi}\dfrac{dx\sin\theta}{r^2}-\hat y\int_R^\infty \dfrac{\mu_0I}{4\pi}\dfrac{dx\sin\theta}{r^2}$$

Como sugerido no exercício, podemos juntar o cálculo dos dois fios em um e retirar a parte central, que nama mais é que usar a superposição das integrais:

$$\vec B=-\hat y\int_{-\infty}^\infty \dfrac{\mu_0I}{4\pi}\dfrac{dx\sin\theta}{r^2}+\hat y\int_{-R}^R \dfrac{\mu_0I}{4\pi}\dfrac{dx\sin\theta}{r^2}$$

Mas o seno do ângulo entre os vetores é dado por:

$$\sin\theta=\dfrac{z}{\sqrt{x^2+z^2}}$$

Voltando pra integral:

$$\vec B=-\dfrac{\mu_0Iz}{4\pi}\hat y\int_{-\infty}^\infty \dfrac{dx}{(x^2+z^2)^{5/2}}+\dfrac{\mu_0Iz}{4\pi}\hat y\int_{-R}^R \dfrac{dx}{(x^2+z^2)^{5/2}}$$

Mas a integral da esquerda nada mais é que o campo magnético de um fio infinito:

$$\vec B=-\dfrac{\mu_0I}{2\pi z}\hat y+\dfrac{\mu_0Iz}{4\pi}\hat y\int_{-R}^R \dfrac{dx}{(x^2+z^2)^{5/2}}$$

Fazendo $x=z\tan\phi\Rightarrow dx=z\sec^2\phi d\phi$, temos:

$$\vec B=-\dfrac{\mu_0I}{2\pi z}\hat y+\dfrac{\mu_0Iz}{4\pi}\hat y\int_{-\arctan{(R/z)}}^{\arctan{(R/z)}} \dfrac{z\sec^2\phi d\phi}{(z^2\tan^2\phi+z^2)^{5/2}}$$

Do princípio fundamental da trigonometria:

$$\sin^2\phi+\cos^2\phi=1\Rightarrow\tan^2\phi+1=\sec^2\phi$$

Então:

$$\vec B=-\dfrac{\mu_0I}{2\pi z}\hat y+\dfrac{\mu_0Iz^2}{4\pi z^5}\hat y\int_{-\arctan{(R/z)}}^{\arctan{(R/z)}} \dfrac{\sec^2\phi d\phi}{(\sec^2\phi)^{5/2}}=-\dfrac{\mu_0I}{2\pi z}\hat y+\dfrac{\mu_0I}{4\pi z^3}\hat y\int_{-\arctan{(R/z)}}^{\arctan{(R/z)}} \cos^3\phi d\phi$$

Novamente pelo princípio fundamental da trigonometria:

$$\vec B=-\dfrac{\mu_0I}{2\pi z}\hat y+\dfrac{\mu_0I}{4\pi z^3}\hat y\int_{-\arctan{(R/z)}}^{\arctan{(R/z)}} (1-\sin^2\phi)\cos\phi d\phi$$

Fazendo $u=\sin\phi\Rightarrow du=\cos\phi d\phi$:

$$\vec B=-\dfrac{\mu_0I}{2\pi z}\hat y+\dfrac{\mu_0I}{4\pi z^3}\hat y\int_{-\frac{R}{\sqrt{R^2+z^2}}}^{\frac{R}{\sqrt{R^2+z^2}}} (1-u^2)du=-\dfrac{\mu_0I}{2\pi z}\hat y+\dfrac{\mu_0I}{4\pi z^3}\hat y \left[u-\frac{1}{3}u^3\right]_{-\frac{R}{\sqrt{R^2+z^2}}}^{\frac{R}{\sqrt{R^2+z^2}}}$$

Ao substituir os limites, ficamos com:

$$\vec B=-\dfrac{\mu_0I}{2\pi z}\hat y+\dfrac{\mu_0I}{4\pi z^3}\hat y \left[\dfrac{2R}{\sqrt{R^2+z^2}}-\frac{2}{3}\dfrac{R^3}{(R^2+z^2)^{3/2}}\right]$$


Temos, portanto:

$$\boxed{\vec B=\hat y\dfrac{\mu_0I}{2\pi z}\left[\dfrac{1}{3z^2}\dfrac{R}{(R^2+z^2)^{3/2}} \left(2R^2+3z^2\right)-1\right]}$$

0
Dislike0
User badge image

RD Resoluções

Nesse exercício vamos usar a Lei de Biot Savart.


Pela lei citada, temos:

$$\vec B=\int \dfrac{\mu_0I}{4\pi}\dfrac{d\vec l\times \hat r}{r^2}$$

Para o nosso caso a corrente está sempre ao longo do eixo $x$:

$$\vec B=-\hat y\int_{-\infty}^{-R} \dfrac{\mu_0I}{4\pi}\dfrac{dx\sin\theta}{r^2}-\hat y\int_R^\infty \dfrac{\mu_0I}{4\pi}\dfrac{dx\sin\theta}{r^2}$$

Como sugerido no exercício, podemos juntar o cálculo dos dois fios em um e retirar a parte central, que nama mais é que usar a superposição das integrais:

$$\vec B=-\hat y\int_{-\infty}^\infty \dfrac{\mu_0I}{4\pi}\dfrac{dx\sin\theta}{r^2}+\hat y\int_{-R}^R \dfrac{\mu_0I}{4\pi}\dfrac{dx\sin\theta}{r^2}$$

Mas o seno do ângulo entre os vetores é dado por:

$$\sin\theta=\dfrac{z}{\sqrt{x^2+z^2}}$$

Voltando pra integral:

$$\vec B=-\dfrac{\mu_0Iz}{4\pi}\hat y\int_{-\infty}^\infty \dfrac{dx}{(x^2+z^2)^{5/2}}+\dfrac{\mu_0Iz}{4\pi}\hat y\int_{-R}^R \dfrac{dx}{(x^2+z^2)^{5/2}}$$

Mas a integral da esquerda nada mais é que o campo magnético de um fio infinito:

$$\vec B=-\dfrac{\mu_0I}{2\pi z}\hat y+\dfrac{\mu_0Iz}{4\pi}\hat y\int_{-R}^R \dfrac{dx}{(x^2+z^2)^{5/2}}$$

Fazendo $x=z\tan\phi\Rightarrow dx=z\sec^2\phi d\phi$, temos:

$$\vec B=-\dfrac{\mu_0I}{2\pi z}\hat y+\dfrac{\mu_0Iz}{4\pi}\hat y\int_{-\arctan{(R/z)}}^{\arctan{(R/z)}} \dfrac{z\sec^2\phi d\phi}{(z^2\tan^2\phi+z^2)^{5/2}}$$

Do princípio fundamental da trigonometria:

$$\sin^2\phi+\cos^2\phi=1\Rightarrow\tan^2\phi+1=\sec^2\phi$$

Então:

$$\vec B=-\dfrac{\mu_0I}{2\pi z}\hat y+\dfrac{\mu_0Iz^2}{4\pi z^5}\hat y\int_{-\arctan{(R/z)}}^{\arctan{(R/z)}} \dfrac{\sec^2\phi d\phi}{(\sec^2\phi)^{5/2}}=-\dfrac{\mu_0I}{2\pi z}\hat y+\dfrac{\mu_0I}{4\pi z^3}\hat y\int_{-\arctan{(R/z)}}^{\arctan{(R/z)}} \cos^3\phi d\phi$$

Novamente pelo princípio fundamental da trigonometria:

$$\vec B=-\dfrac{\mu_0I}{2\pi z}\hat y+\dfrac{\mu_0I}{4\pi z^3}\hat y\int_{-\arctan{(R/z)}}^{\arctan{(R/z)}} (1-\sin^2\phi)\cos\phi d\phi$$

Fazendo $u=\sin\phi\Rightarrow du=\cos\phi d\phi$:

$$\vec B=-\dfrac{\mu_0I}{2\pi z}\hat y+\dfrac{\mu_0I}{4\pi z^3}\hat y\int_{-\frac{R}{\sqrt{R^2+z^2}}}^{\frac{R}{\sqrt{R^2+z^2}}} (1-u^2)du=-\dfrac{\mu_0I}{2\pi z}\hat y+\dfrac{\mu_0I}{4\pi z^3}\hat y \left[u-\frac{1}{3}u^3\right]_{-\frac{R}{\sqrt{R^2+z^2}}}^{\frac{R}{\sqrt{R^2+z^2}}}$$

Ao substituir os limites, ficamos com:

$$\vec B=-\dfrac{\mu_0I}{2\pi z}\hat y+\dfrac{\mu_0I}{4\pi z^3}\hat y \left[\dfrac{2R}{\sqrt{R^2+z^2}}-\frac{2}{3}\dfrac{R^3}{(R^2+z^2)^{3/2}}\right]$$


Temos, portanto:

$$\boxed{\vec B=\hat y\dfrac{\mu_0I}{2\pi z}\left[\dfrac{1}{3z^2}\dfrac{R}{(R^2+z^2)^{3/2}} \left(2R^2+3z^2\right)-1\right]}$$

0
Dislike0

Faça como milhares de estudantes: teste grátis o Passei Direto

Esse e outros conteúdos desbloqueados

16 milhões de materiais de várias disciplinas

Impressão de materiais

Agora você pode testar o

Passei Direto grátis

Você também pode ser Premium ajudando estudantes

✏️ Responder

SetasNegritoItálicoSublinhadoTachadoCitaçãoCódigoLista numeradaLista com marcadoresSubscritoSobrescritoDiminuir recuoAumentar recuoCor da fonteCor de fundoAlinhamentoLimparInserir linkImagemFórmula

Para escrever sua resposta aqui, entre ou crie uma conta

User badge image

Outros materiais